Wednesday, January 1, 2014

Principle Question Problem Set Answer Key

Question #1. Must-PR. October 2002 LSAT, Section 1, #7. The correct answer choice is (D)
The principle in the stimulus is clearly stated: “one should always have one’s own work checked by
someone else.” Your task is to find the answer that most closely follows that guideline.
Answer choice (A): This answer choice does not contain any work to be checked, and therefore it cannot
illustrate the principle in the stimulus.
Answer choice (B): Although this answer discusses “one’s view,” a view is not the same as “one’s own
work” and therefore the principle cannot be applied to this answer.
Answer choice (C): Although this answer involves the checking of work (juries “check” the work of
lawyers), the heart of this answer is not that one should have one’s own worked checked by others. Rather,
this answer focuses on who is in the best position to do the checking: experts or novices.
Answer choice (D): This is the correct answer. The answer matches the principle by stating that a writer
should have his or her work proofread by others, and then further provides a justification that matches the
premise in the stimulus (that other people are better at detecting errors).
Answer choice (E): Ordering a meal does not qualify as “one’s own work,” nor does eating the meal
qualify as checking the work. Hence, this answer has no attribute that illustrates the principle.
Overall, this question is classified as relatively easy.
Question #2. Strengthen-PR. October 2003 LSAT, Section 1, #17. The correct answer choice is (C)
The stimulus presents a clear and easy-to-understand situation, and Sharon’s actions can be described as
follows: Sharon paid the asking price for an item even though she was fully aware the asking price was far
too low, and she feels her actions are not inappropriate.
The question stem requires you to identify the answer that contains a principle that would most strengthen
Sharon’s position regarding her actions. The principle that supports Sharon’s position must address, in
some way, the discrepancy between what Sharon paid for the item and what she knew to be the value of
the item.
Please note that your personal view of the ethics of Sharon’s actions have no place in this question. The
stimulus describes what she did, and the nature of the question forces you to identify the answer that best
justifies her decision.
Answer choice (A): This answer focuses on the obligations of the seller, not the buyer. Since Sharon is the
buyer and it is her actions that are under scrutiny, this answer choice does not apply to the scenario in the
stimulus. Remember, read carefully! This is a Reverse answer, and if you switch buyer and seller in the
answer it will be correct.
Answer choice (B): The issue in the stimulus is not whether Sharon paid the storekeeper the asking
price—she did. The issue is whether she took advantage of him by not disclosing the fact that the item was
worth more than the asking price. Since this answer focuses on the whether the amount of money paid
matches the asking price, it is incorrect.
Answer choice (C): This is the correct answer. The principle in this answer makes it clear that a buyer has
no obligation beyond paying the asking price for an item. If this is true, then Sharon is justified in not
disclosing the true value of the item to the storekeeper.
Answer choice (D): Sharon’s position revolves around the discrepancy between what Sharon paid for the
item and what she knew to be the value of the item. This answer choice addresses the buyer’s
responsibility regarding the quality of the item, and does not touch on the issue of the discrepancy.
Note that it appears Sharon did ascertain that the quality of the item met her standards. Regardless, since
the principle fails to address the defensibility of her position, the answer is incorrect.
Answer choice (E): There is no indication that Sharon and the storekeeper were “well acquainted,” so this
principle cannot be applied to the situation in the stimulus. If they were well acquainted, then Sharon’s
position would be weakened.
Question #3. Must-PR, SN. December 2002 LSAT, Section 1, #11. The correct answer choice is (D)
Statistically, this question is considerably more difficult than either of the previous two questions. The
difficulty occurs because many students mis-diagram one or both of the conditional statements in the
stimulus. Although the question stems asks you to identify the answer that follows from the principle, the
principle contains two similar but distinct conditional statements, you must be aware that either could
figure in the correct answer. We will now examine both of the statements (the conditional indicators are
italicized):
1. “A gift is not generous unless it is intended to benefit the recipient and is worth more than what is
expected or customary in the situation.”
Applying the Unless Equation, the phrase modified by “unless” becomes the necessary condition
and the remainder is negated and becomes the sufficient condition:
G = a gift is generous
B = the gift is intended to benefit the recipient
WM = the gift is worth more than what is expected or customary in the situation
             B
G----------> +
            WM
As discussed in the chapter, the application of a conditional principle similar to this one does not
allow a conclusion to be drawn that someone is generous (however, a conclusion could be made
via the contrapositive that someone is not generous).
2. “A gift is selfish if it is given to benefit the giver or is less valuable than is customary.”
Remember, “if” introduces a sufficient condition.
S = a gift is selfish
BG = the gift is given to benefit the giver
LV = the gift is less valuable than is customary
BG
or-----------> S
LV
Some students compare the two statements in the stimulus and conclude that the second is the
contrapositive of the first. While the second relationship is very similar to the contrapositive of the first
relationship, it is not an exact contrapositive because the logical opposite of “generous” is not “selfish” (it
is “not generous”), the logical opposite of “benefit the recipient” is not “benefit the giver” (it is “not benefit
the recipient”), and the logical opposite of “worth more than customary” is not “worth less than
customary” (it is “not worth more than customary”).
As you examine the answer choices, keep in mind that the test makers will try to devise answer choices
that are similar to the guidelines in the principle, but that do not match exactly. These answers will be
incorrect. Remember also that the contrapositive of either conditional statement can be used to arrive at the
correct answer.
Answer choice (A): This is a very attractive wrong answer choice.
To draw the conclusion that one’s gift is selfish, one must either give a gift that is given to benefit the giver
or give a gift that is less valuable than is customary. Let us examine the answer choice and determine
whether Charles’s gift meets either condition:

1. Was the gift given to benefit the giver?
No. Even though Charles hates opera, giving the tickets to his cousin did not benefit Charles
(Charles was under no apparent obligation to attend the opera and could have left the tickets
unused). In fact, they clearly benefitted his cousin, who loves opera.
2. Was the gift less valuable than is customary?
Unknown. Even though Charles paid nothing for the tickets, they still had value (they could have
been resold, for example). Whether they were less valuable than customary for a birthday gift is
unknown (although common sense suggests they were not less valuable).
Since neither sufficient condition has been met with certainty, it cannot be concluded that Charles’s gift
was selfish.
Answer choice (B): Again, does the answer choice meet either sufficient condition?
1. Was the gift given to benefit the giver?
No, the gift was given in order to help keep Emily’s brother healthy.
2. Was the gift less valuable than is customary?
No, the gift was apparently given for no special occasion.
In any scenario under this principle, the consequences of the gift (in this case that Emily’s brother was hurt
and offended) do not play a role in determining whether the gift was a selfish one.
Answer choice (C): This answer concludes that Amanda is generous on the basis of her actions. As
discussed previously, there is no way to use the principle to conclude that an individual is generous.
Once you see the answer concluding that a sufficient condition (“generous”) occurred, you can eliminate
the answer with speed and confidence. Simply put, structurally an answer of this type could never be
correct in a problem such as this one.
Answer choice (D): This is the correct answer. In order to arrive at the judgment that a gift is not generous,
one or both of the necessary conditions from the first conditional relationship must not be met. Does that
occur in this answer?
1. Was the gift intended to benefit the recipient?
Yes. This gift was intended to benefit the recipient, and so this necessary condition was met.
2. Was the gift worth more than what is expected or customary in the situation?
No. The answer choice indicates that all the children in Olga’s family receive a computer for
graduation. Since the gift of a computer is not worth more than is customary in the situation, this
condition is not met and via the contrapositive we can conclude that Olga’s gift was not generous.
Since one of the necessary conditions in the principle was not met, we can conclude that the sufficient
condition was not met, and Olga’s gift was not a generous one. As this matches the judgment in the answer
choice, this is the correct answer.
Answer choice (E): Since the judgment in this answer (“not generous”) is the same as in answer choice
(D), we can analyze this answer in the same way:
1. Was the gift intended to benefit the recipient?
Yes. Michael gave his nephew $50 dollars, and in doing so the intention was to benefit the
nephew. The fact that the nephew subsequently lost the money does not show that Michael’s
intention was not to benefit his nephew. To some extent, this situation is like the one in answer
choice (B) because in the principle the consequences of the gift do not play a role in determining
the intentions of the giver.
2. Was the gift worth more than what is expected or customary in the situation?
Yes, the first line spells out that the gift of $50 was more than Michael had ever given his nephew
before.
Since both necessary conditions of the principle are met, the gift could be a generous one, a conclusion
that does not conform to the judgment in the answer choice. Hence, the answer is incorrect.
A final note on this question: make sure you are comfortable with how the conditional relationship in this
Must-PR problem (and in the one in the chapter text) is used to affect the answer choices (whether you can
determine if someone is generous or not generous, selfish or not selfish). This form of Principle question
has proven to be hard enough that the test makers will surely continue to place this type of question on the
exam.
Question #4. Strengthen-PR. December 2003 LSAT, Section 2, #15. The correct answer choice is
(B)
The argument in the stimulus is composed as follows:
Premise: Some scientists have expressed reservations about quantum theory because of its
counterintuitive consequences.
Premise: But despite rigorous attempts to show that quantum theory’s predictions were
inaccurate, they were shown to be accurate within the generally accepted statistical
margin of error.
Conclusion: These results, which have not been equaled by quantum theory’s competitors, warrant
acceptance of quantum theory.
The argument uses the “some people say” construction discussed in Chapter Two. As some scientists are
said to have reservations about quantum theory, you can predict that the author will conclude that they
should not have these reservations, and indeed this occurs in the last sentence of the stimulus.
The author’s reason for concluding that quantum theory should be accepted is that all attempts to disprove
the theory have failed. From the Flaw in the Reasoning section we know this is an error in the use of
evidence: lack of evidence against a position is mistakenly taken to prove that position is true. The
question stem asks you to identify a principle that could help to justify this form of erroneous reasoning.
Answer choice (A): The author’s reasoning is not based on “fewer counterintuitive consequences,” but on
the fact that the consequences have not been disproven. Hence, this principle would not help the reasoning
and this answer is incorrect.
Answer choice (B): This is the correct answer choice. If this principle is accepted, then the argument in the
stimulus is strengthened. The principle, when slightly restated, asserts that “if a scientific theory has been
subjected to serious attempts to disprove it and has withstood all of them, then it should be accepted.” We
know from the stimulus that the theory has been subject to serious attempts to disprove it and it has
withstood them. According to the principle, this justifies the conclusion that the theory should be accepted.
Answer choice (C): The conclusion of the argument in the stimulus is that the theory should be accepted.
Using the principle in this answer choice would lead one to conclude that the consequences of the theory
should not be considered counterintuitive. Since this result differs from the conclusion, this answer choice
is incorrect.
Answer choice (D): This answer, when rephrased using the Unless Equation, reads “if a theory should be
rejected, then it has been subjected to serious attempts to disprove it.” The scenario in the stimulus meets
the necessary condition, but that has no impact on whether the sufficient occurs or does not occur.
Answer choice (E): This is the Mistaken Reversal of answer choice (B). Applying this principle fails to
justify the argument since only the necessary condition is met, and that does not prove the sufficient
condition (in this case, that the theory should be accepted) will occur. Hence, this answer choice is
incorrect.

No comments:

Post a Comment